MCAT Psych/Soc

Ace your homework & exams now with Quizwiz!

Solomon Asch Experiment

Conforming to the group (line test)

operational definition

Creating an operational definition for the dependent variable would be important in experimental design

Mental Disorders (Categories in DSM-5)

DSM-5 is the American Psychiatric Association's Diagnostic and Statistical Manual of Mental Disorders, Fifth Edition; a widely used system for classifying psychological disorders. (Level 1 - categories) (Level 2 - categories and specific diagnoses)

Cannon-Bard Theory of Emotion

Emotion and physiological reactions occur simultaneously

Gender vs sex

Gender is a social construct; men and women are socialized to be masculine and feminine. Sex is a biological measure; men and women are born male or female based on biological differences, such as their anatomic genital differences

Classical conditioning (generalization vs discrimination)

Generalization: the tendency, once a response has been conditioned, for stimuli similar to the conditioned stimulus to elicit similar responses Discrimination: in classical conditioning, the learned ability to distinguish between a conditioned stimulus and stimuli that do not signal an unconditioned stimulus

General Adaptation Syndrome (GAS)

Hans Seyle's concept that the body responds to stress with alarm, resistance and exhaustion

What area of the brain is central to memory and learning?

Hippocampus! If you see a hippo on campus, you'll remember it!

Multitasking/divided attention (successful multitasking depends on what)

If one must multitask, it would be best to choose an auditory and visual task

internal vs external validity

Internal validity describes the extent that a study is able to show a cause-effect relationship between the variables tested in the study. The extent to which the outcome variable is due to the intervention. How well done a study is or how well it allows researchers to draw conclusions from measured effects. External validity describes the extent that the results of a study can be generalized or repeated in multiple settings.

Conformiity

Involves changing one's behavior in order to fit in with the norms of a particular social group, most typically a group that has a certain level of social importance.

Compliance

Involves changing one's behavior in response to a request from another person who is of equal or lower status (thus differing from obedience).

Obedience

Involves changing one's behavior in response to the direct command or order of a person who is in authority or is of higher social status.

spreading activation

Occurs when one item brought into working memory triggers an activation of related memory

micro sociology

Premise: human behavior is the result of individual interpretations. Focuses on the smallest building blocks of society and builds up to larger structures Begins with one-on-one interactions Bottom-up view

Elaboration likelihood model

Proposes two cognitive routes of persuasion 1. In the central route, people are persuaded by the content of the argument itself. 2. In the peripheral route, people focus on superficial or secondary characteristics of the speech or the orator, like the attractiveness of the orator, the length of the speech, whether the orator is considered an expert in his field, etc.

What kind of thinkers were Erik Erikson and Sigmund Freud?

Psychoanalytic thinkers

Hypothalmic-Pituitary-Adrenal (HPA) Axis

Responds to perceived stressors by initiating the release of cortisol

Self concept

Social attributes can be easily remembered using the ADDRESSING framework: Age, Disability status (physical), Disability status (mental), Religion, Ethnicity/race, Sexual orientation, Socioeconomic status, Indiginous background, Nation of origin, and Gender.

Ethnocentrism

The belief in the inherent superiority of one's own ethnic group or culture.

optimism bias

The belief that bad things happen to other people but not to us

Behaviorist perspective

Theories of personality

Malthusian Theory

Thomas Malthus hypothesized that unchecked population growth would quickly exceed carrying capacity, leading to overpopulation and catastrophes such as famine or war (known as a ______ catastrophe)

type 1 vs type 2 error

Type 1: false positive (rejecting the null hypothesis when in fact it is true) Type 2 : false negative (accepting the null hypothesis when in fact it is false) (i.e., concludes that a relationship or effect does not exist when it actually does)

Validity vs. Reliability

Validity: does it do what it's supposed to do? does it work? Reliability: the degree to which the tool produces consistent/stable results Validity: measure of adequacy Reliability: measure of consistency

7 senses

Vision (seeing), Auditory (hearing), Olfactory (smell), Tactile (touch), Gustatory (taste), Proprioception (body awareness), Vestibular (balance)

feature detection theory

We activate different areas of the brain when looking at different features of an image. Allows for parallel processing of a visual stimulus.

Weber's Law

Weber's Law states that, in order for humans to perceive sensory differences: two stimuli must differ by a constant proportion, which varies by the type of stimulus, but remains constant within a given stimulus. Just noticeable difference: (Light = 8%, sound = 0.3%, weight = 2%)

Optimal arousal theory/Yerkes-Dodson law

Yerkes-Dodson law refers to how stress and arousal affect performance; a moderate level of stress helps people perform at their best

Schema

a concept or framework that organizes and interprets information

Consciousness brain wave patterns

alpha, beta, that, delta

learned helplessness

an individual possesses low self-efficacy and an external locus of control.

Baddeley's model of working memory

dominant theory in the field of working memory. Consists of the central executive which acts as supervisory system and controls the flow of information from and to its slave systems: the phonological loop, visuo-spatial sketchpad, and the episodic buffer

monozygotic twins vs dizygotic twins

identical vs fraternal -Mono zygotic: identical twins share 100% of DNA -Dizygotic twins: non identical twins share 50% of DNA -monozygotic twins are developed from one embryo, splitting into two embryos whereas dizygotic twins are developed from two different eggs, which are fertilized by sperms separately

Desensitization

involves trying to no longer respond to a stimulus that one was once sensitized to

Changes in synaptic connections in the brain

neural plasticity and long term potentiation

Stanley Milgram Experiment

obedience to authority

Aversive control

occurs when behavior is motivated by the reality or threat of something unpleasant happening (unpleasant ... can be benign or life threatening)

Perceptual constancy

perceiving objects as unchanging (having consistent lightness, color, shape, and size) even as illumination and retinal images change

normative influence

refers to a group effect that results from individuals' desire to be liked and to win the approval of others

Retina Photoreceptors

rods and cones -blind spot has no photoreceptors

Role (role conflict, role strain, role exit)

socially defined expectations based on a position in society

Encoding and encoding strategies

the processing of information into the memory system

fundamental attribution error

the tendency for observers, when analyzing others' behavior, to underestimate the impact of the situation and to overestimate the impact of personal disposition

Token economy

uses secondary reinforcers

What is the estimated capacity and duration of our short-term memory stores? A. 15-20 item capacity; 15-30 second duration B. 5-9 item capacity; 15-30 second duration C. 15-20 item capacity; 1-5 second duration D. 5-9 item capacity; 1-5 second duration

(Class 6: FSQ Neuroscience, Memory, and Learning) Our short-term memory stores are estimated to have a roughly 5-9 item capacity (roughly seven items plus or minus two) with an approximately 15-30 second duration.

Major demographic factors in society

(Two major theories of demographic change: Malthusian Theory and Demographic Transition Model)

Drive-reduction theory

(motivation)

Circadian rhythm

- biological clock; regular bodily rhythms that occur on a 24-hour cycle -The three physiological indicators of a mammal's circadian rhythm are melatonin released by the pineal gland, body temperature, and serum cortisol levels.

androcentric culture

-A culture that is male-oriented; the male is considered the norm and is consistently preferred across the various aspects of the culture

Social construct

-Anything that appears natural or obvious to the people that accept it but is largely the invention of a given society -An idea that has been created and accepted by the people in a society -One example of is "childhood." While there are certain biological differences between children and adults, different societies and different times have conceptualized childhood in very different ways. -Social constructs do not have inherent meaning. The only meaning they have is the meaning given to them by people. For example, the idea that pink is for girls and blue is for boys is an example of a social construct related to gender and the color of items.

class consciousness vs false consciousness

-Class consciousness is a social condition in which members of a subordinate social class are actively aware of themselves as a group that is exploited by the wealthy. -According to Marx, false consciousness is a lack of such awareness; this occurs when members of a subordinate class see themselves as individuals instead of as an exploited group.

Habituation

-Decreasing responsiveness with repeated stimulation -An organism's decreasing response to a stimulus with repeated exposure to it

Self esteem

-Generalized view of oneself; a BELIEF about how good or bad a person you are overall -Belief about one's self worth

Group polarization

-Group that agrees on a topic discusses it together -The belief of the group becomes more extreme -group polarization is NOT when a group becomes more divided on an issue

Popular culture

-Includes all of the ideas, perspectives, attitudes, images, and other phenomena that are within the mainstream of Western culture due to the emerging global mainstream of the late 20th and early 21st century. -Heavily influenced by mass media, this collection of ideas permeates the everyday lives of the individuals in a society.

Deindividuation

-Large group participating in an arousing activity -Loss of individual identity, development of a 'mob mentality'

Dishabituation

-Recovery of a habituated response after a change in stimulation -The restoration to full strength of a response to a stimulus that had previously become weakened through habituation -Increase in responsiveness after presentation of a new stimulus

Sensation vs. Perception

-Sensation is arrival of info to brain, perception is interpretation of info in brain -Sensation is the stimulation of the sense. Perception is how the brain interprets the stimulus

Bystander effect

-Someone needs help -If many others are present, everyone is less likely to help

Cultural transmission

-The methods a group of people within a society or culture use to learn and pass on new information. -The key aspect of culture is that it is not passed on biologically from the parents to the offspring, but rather learned through experience and participation.

Assimilation

-The process by which a person or a group's language and or culture come to resemble those of another group. (e.g. a minority group's adoption of the beliefs and way of life of the dominant culture)

Insight learning

-a form of problem solving in which the organism develops a sudden insight into or understanding of a problem's solution -the process of mentally working through a problem until the sudden realization of a solution occurs

dispositional attribution vs situational attribution

-attributing behavior to the person's disposition and traits -attributing behavior to the environment

nepotism

-favoritism to relatives -getting favorable treatment due to personal family connections

latent learning

-learning that remains hidden until its application becomes useful -learning that occurs but is not apparent until there is an incentive to demonstrate it -knowledge that only becomes clear when a person has an incentive to display it. For example, a child might learn how to complete a math problem in class, but this learningis not immediately apparent. Only when the child is offered some form of reinforcement for completing the problem does this learning reveal itself.

The five major functions of family

-reproduction and monitoring of sexual behaviors -social status -affection and companionship -socialization -protection

just-world hypothesis

-the assumption that the world is fair and that therefore people get what they deserve and deserve what they get -the belief that people get what they deserve in life and deserve what they get

actor-observer bias

-when we blame our own actions on the situation and blame the actions of others on their personalities -the tendency to attribute one's own behavior to situational factors but to attribute the behavior of others to dispositional factors -the tendency to blame our actions on the situation and blame the actions of others on their personalities

Bipolar 1 vs Bipolar 2

1 = Man"I"a 2 = Depre"ss"ion

Discrimination vs prejudice

= ACT against a person or people based on negative beliefs about the group to which that person or people belong. = negative BELIEF about a group of people

aggregate vs category

A group is a number of people (as few as two) who identify and interact; -an aggregate includes people who exist in the same space, but do not identify or interact -a category shares certain characteristics, but does not reqularly interact.

Altruism

Altruism occurs when an individual helps someone else despite incurring a cost or risk for helping. Large crowds of witnesses actually reduce the likelihood that altruistic behavior will occur

Sensitization

An increase in behavioral response after exposure to a stimulus

James-Lange Theory of Emotion

Arousal causes emotion

Self schema

Beliefs about the self

Albert Bandura

Bobo doll experiment

Broca vs Wernicke aphasia

Broca - area of speech production, so aphasia causes difficulty producing speech Wernicke - area of speech comprehension, so aphasia causes difficulty understanding speech, so people will talk in sentences that don't make sense

Availability heuristic

estimating the likelihood of events based on their availability in memory; if instances come readily to mind (perhaps because of their vividness), we presume such events are common

Multiculturalism

The preservation of various cultures or cultural identities within a single unified society.

Cultural relativism

The principle that an individual human's beliefs and activities should be understood by others in terms of that individual's own culture.

A student is just able to perceive that a black, circular dot on a sheet of paper is larger than another dot when the first dot is 100 mm across and the second is 115 mm across. According to Weber's law, if the same student is presented with a black, circular dot that is 50 mm across, which of the following is the diameter of a dot that the student would just be able to perceive as being larger? A. 35 mm B. 42.5 mm C. 57.5 mm D. 65 mm

(Blueprint)

null hypothesis and p-value

The smaller the p-value, the stronger the evidence that you should reject the null hypothesis. A p-value less than 0.05 (typically ≤ 0.05) is statistically significant. It indicates strong evidence against the null hypothesis, as there is less than a 5% probability the null is correct (and the results are random).

Cultural diffusion

The spread of cultural beliefs and social activities from one group to another.

halo effect

refers to a tendency to attribute many positive attributes to someone based on a single positive attribute. For example, we may think someone is kind, smart, and generous simply because they are attractive. The halo effect helps to explain the fact that we tend to think attractive people are good and trustworthy. The halo effect is a tendency to believe that people have inherently good or bad natures, rather than looking at individual characteristics. The physical attractiveness stereotype is a specific type of halo effect; people tend to rate attractive individuals more favorably for personality traits and characteristics than they do those who are less attractive.

McDonalidization

refers to principles of the fast-food industry dominating other sectors of society.

mere exposure effect

when repeated exposure to a person increases familiarity, which in turn increases fondness

Biological Perspective

Theories of personality

self-serving bias

the tendency to perceive oneself favorably

The tendency of a new mother to actively join networks almost exclusively with other young mothers is an example of: A. implicit bias. B. confirmation bias. C. heterophily. D. homophily.

(Blueprint) Answer is choice D! -Implicit bias refers to the attitudes or stereotypes that affect an individual's understanding, actions, and decisions in an unconscious manner. These biases, which can encompass both favorable and unfavorable assessments, are activated involuntarily and without an individual's awareness. -Confirmation bias is the tendency of an individual to search for, interpret, or recall information in a way that confirms their existing beliefs or hypotheses. -Heterophily (the opposite of homophily) is the tendency of individuals to collect in diverse groups. -Homophily is the tendency of individuals to associate and bond with others similar to themselves.

A physician accepts a new job and finds that for the first several months of work, most of their effort is spent learning the mechanisms by which doctors, patients, insurance companies, and the government interact to deliver and pay for care. They particularly note that the influence by the hospital's legal department often supersedes that of other important administrators who hold medical degrees. Their experiences and observations MOST closely reflect what theory of sociology? A. Functionalism B. Conflict theory C. Feminist theory D. Symbolic interactionism

(Blueprint) Correct answer is A. -Functionalism, one of the sociological approaches you should understand for the MCAT, views society and social institutions as interdependent, interacting parts of a whole. Here, the various elements of the hospital system come together to function as a system to deliver health care to the patients. -This choice is tempting! Conflict theory, pioneered by Karl Marx, views sociology through the lens of conflicts between various classes and groups within society. Often, this involves systematic inequalities that keep power out of the hands of lower-status groups. While the question stem does mention that the legal department holds more influence than some administrators, this alone does not constitute a conflict; overall, the different parts of the hospital are depicted as working together rather than clashing. In addition, administrators with advanced medical degrees would not constitute a lower-status group. -Feminist theory is a branch of feminism that seeks to explain the nature of gender inequality. It examines women's social roles, experience, interests, and politics in a variety of fields such as education, medicine, and business. Although the physician mentioned is female, nothing in the question's description specifically relates to her role in the hospital system as a woman. -Symbolic interactionism is a view of sociology on the micro level, through interactions between individual people. It explains how, as members of society, those individuals use and understand the symbols of the society they live in. The question stem does not focus on symbols.

One study participant described overeating in response to a sense of anxiety about the state of the world, especially in relation to global climate change and the poor economy. This anxiety most closely reflects the effect of which kind of stressor? A. Acute B. Microstressor C. Crisis D. Ambient

(Blueprint) Correct answer is D. -An acute stressor is one that is present over a short period of time. This does not apply to a persistent issue like the state of the economy. -Microstressors are small daily hassles like dealing with traffic or irritating coworkers. -A crisis is a sudden, rare occurrence such as a natural disaster or war that causes a person a great deal of stress. -Ambient stressors are those chronic environmental stressors that cannot be changed (or are perceived as unchangeable) by the efforts of the individual subject to them. Things like the economy and climate change would be examples of ambient stressors.

Of the following types of disorders, which is the most prevalent in the U.S? A. Mood disorders B. Anxiety disorders C. Personality disorders D. Psychotic disorders

(Class 5: FSQ Psychological Disorders and Consciousness) Anxiety disorders (panic disorder, obsessive-compulsive disorder, generalized anxiety disorder, and phobias) are prevalent in about 18% of the U.S. population. Mood disorders (major depressive disorder, dysthymic disorder, and bipolar disorder) are prevalent is about 9.5% of the U.S. population (choice A is wrong). Personality disorders are prevalent in about 9% of the U.S. population (choice C is wrong), and psychotic disorders are prevalent in about 2% of the U.S. population (choice D is wrong).

In order to be properly labeled a "psychological disorder," behavior must be: A. maladaptive, statistically unusual, characterized by cognitive or perceptual distortion, and considered abnormal in the society in which it occurs. B. statistically unusual, considered abnormal in most societies, characterized by cognitive or perceptual distortion, and maladaptive. C. characterized by cognitive or perceptual distortion, maladaptive, statistically unusual, and amenable to treatment. D. considered abnormal in most societies, statistically unusual, maladaptive, and characterized by cognitive or perceptual distortion.

(Class 5: FSQ Psychological Disorders and Consciousness) In order to be correctly labeled "disordered," behavior must meet four criteria. First, the behavior must be maladaptive, i.e., it must impair one's ability to function in one or more situations (all four answer choices list this criterion). Second, the behavior must be statistically unusual; all four answer choices list this criterion as well. Third, the behavior must be characterized by cognitive or perceptual distortion; all four choices list this criterion. Fourth, the behavior must be considered abnormal in the society in which it occurs. The behavior need not be considered abnormal in most societies, as cultural norms differ greatly (choices B and D can be eliminated). Choice C lists "amenable to treatment" as a criterion and there is no such requirement; a psychological disorder can exist even if it cannot be effectively treated (choice C can be eliminated and choice A is correct).

Dependence on alcohol: I.) is caused in part by dopamine release from the nucleus accumbens. 2.) means that, upon cessation of use, withdrawal symptoms will occur. 3.) may lead to down-regulation of inhibitory pathways in the central nervous system. A. I and III only B. I and II only C. I, II, and III D. II and III only

(Class 5: FSQ Psychological Disorders and Consciousness) Item I is true: dopamine release in the nucleus accumbens (often called the "reward center" of the brain) is likely critical for the development of addiction to alcohol (choicd D can be eliminated). Item II is true: dependence can lead to signs of withdrawal upon abstention from alcohol (choice A can be eliminated). Item III is true: development of dependence (also known as addiction) to alcohol (a central nervous system depressant) involves changes in brain chemistry with long-term use, such as a down-regulation of inhibitory pathways in the central nervous system (choice B can be eliminated and choice C is correct).

General fertility rate vs Replacement level fertility vs Crude birth rate vs Fertility

-General fertility rate refers to the annual number of live births for every 1,000 women of childbearing age. -Replacement level fertility is the number of offspring that must be produced in order to replace people who have died in a given population. -The crude birth rate is the annual number of live births per 1,000 people in a given population, as opposed to specifically women of childbearing age. -In demographics, fertility is defined simply as a women's ability to give birth to children, not the number of children per 1,000 women.

Obsessive-compulsive disorder is often characterized by: I.) repeated, intrusive, uncontrollable thoughts or impulses that cause distress or anxiety. II.) repeated physical or mental behaviors that are done in accordance with a set of strict rules, in order to reduce distress or prevent something dreaded from occurring. III.) psychotic episodes triggered by high anxiety. A. I and III only B. I, II, and III C. I and II only D. II and III only

(Class 5: FSQ Psychological Disorders and Consciousness) Item III is false: obsessive-compulsive disorder (OCD) does not include psychotic episodes (choices A, B, and D can be eliminated and choice C is correct). Note that Item I is true: a person with OCD has obsessions, compulsions, or both. Obsessions are repeated, intrusive, uncontrollable thoughts or impulses that cause distress or anxiety. The person knows the thoughts are irrational, and tries to disregard or suppress them, or to neutralize them through a compulsive behavior. Item II is also true: compulsions are repeated physical or mental behaviors that are done in response to an obsession or in accordance with a set of strict rules, in order to reduce distress or prevent something dreaded from occurring. The person realizes that the compulsive behavior is not reasonable, being either unrelated to the dreaded event, or related but clearly excessive. Nevertheless, if the person does not perform the behavior, he or she feels intense anxiety and a conviction that the terrible event will happen.

Mindfulness meditation: A. induces delta brain waves. B. is associated with improved concentration and lower blood pressure. C. creates a state of high suggestibility in the subject. D. is often used by psychotherapists to target specific unwanted behaviors, such as smoking or phobic reactions.

(Class 5: FSQ Psychological Disorders and Consciousness) Mindfulness meditation is associated with improved overall concentration and lower blood pressure (choice B is correct). It does not induce delta brain waves; it is associated with lower frequency alpha and theta waves (choice A is wrong). Hypnosis, not meditation, creates a state of high suggestibility in the subject. Consequently, unlike meditation, it is often used by psychotherapists to eliminate unwanted behaviors through posthypnotic suggestion (choices C and D are wrong).

All of the following true of schizophrenia EXCEPT: A. schizophrenia is a personality disorder characterized by both positive and negative symptoms. B. schizophrenia is typically diagnosed in the late teens, 20s or early 30s. C. both genes and environment are thought to contribute to the development of schizophrenia. D. schizophrenia is a psychotic disorder that affects approximately 1% of the population. Schizophrenia is not a personality disorder, but rather a psychotic disorder, that is characterized by both positive and negative symptoms. Schizophrenia is typically diagnosed during early adulthood; anywhere from the late teens to early 30s, though diagnoses have been made much earlier and later. Both genes and environment are thought to contribute to the development of schizophrenia and it affects roughly 1% of the population. Since choice A stands out with an "F" instead of a "T", it is the correct answer choice.

(Class 5: FSQ Psychological Disorders and Consciousness) Schizophrenia is not a personality disorder, but rather a psychotic disorder, that is characterized by both positive and negative symptoms. Schizophrenia is typically diagnosed during early adulthood; anywhere from the late teens to early 30s, though diagnoses have been made much earlier and later. Both genes and environment are thought to contribute to the development of schizophrenia and it affects roughly 1% of the population. Since choice A stands out with an "F" instead of a "T", it is the correct answer choice.

Which of the following would be the best indicator that Jacques has somatic symptom disorder? A. He frequently worries about developing illnesses that the doctor says Jacques currently does not have. B. He frequently complains of experiencing pain. C. His health is his primary concern and he discusses it daily. D. His fear of illness, which is excessive and largely unfounded, greatly impairs his ability to function.

(Class 5: FSQ Psychological Disorders and Consciousness) Somatic symptom disorder (previously known as hypochondriasis) is characterized by exaggerated fear of illness and irrational worrying about one's health. This preoccupation must cause the individual distress or impair his ability to function. The fact that Jacques frequently worries about illnesses he doesn't currently have doesn't necessarily mean that his level of fear is excessive or that he is distressed or impaired by this preoccupation (choice A is wrong). If Jacques often complains about pain, he may in fact be experiencing considerable pain (choice B is wrong). Similarly, if Jacques's primary concern is his health, this preoccupation may in fact be justified and realistic (choice C is wrong and choice D is correct).

A psychiatrist who uses a biomedical approach to treating mental disorders is performing an intake on a new patient. The doctor would probably be most interested in which of the following pieces of information about this individual? A. Socioeconomic status B. Cultural beliefs concerning illness C. Family history of schizophrenia D. Quality of peer relationships

(Class 5: FSQ Psychological Disorders and Consciousness) The biomedical approach views mental disorders as diseases and is concerned primarily with medical information, such as a possible genetic predisposition to schizophrenia (choice C). The biopsychosocial approach, in contrast, takes many more aspects of the patient's life into account, including sociocultural and behavioral determinants. A clinician using the biopsychosocial approach would be very interested in socioeconomic status, cultural beliefs, and peer relationships, but a biomedical clinician would not (choices A, B, and D are wrong and choice C is correct).

Alertness and arousal are controlled by: A. the cerebellum. B. the thalamus. C. the pons. D. the reticular activating system (RAS).

(Class 5: FSQ Psychological Disorders and Consciousness) The reticular activating system (RAS), which is located in the midbrain, controls alertness and arousal. The cerebellum is located behind the brain and is an integrating center where complex movements are coordinated (choice A is wrong). The thalamus is located near the middle of the brain, below the cerebral hemispheres and above the midbrain, and contains relay and processing centers for sensory information (choice B is wrong). The pons is located below the midbrain and controls certain autonomic functions, such as breathing and swallowing (choice C is wrong).

Maria experiences various circadian rhythm changes as she travels on an airplane for twenty-four hours. Which of the following is NOT a measurable physiological indicator of these changes? A. Melatonin levels B. Serum cortisol levels C. Dopamine levels D. Body temperature

(Class 5: FSQ Psychological Disorders and Consciousness) The three physiological indicators of circadian rhythms are melatonin (which controls sleepiness), serum cortisol (which controls glucose metabolization) and body temperature. While there is some evidence that dopamine affects the circadian cycle, it is NOT a measurable indicator of circadian rhythm changes (choice C is the correct answer choice).

The General Adaptation Syndrome theorizes that: A. when a stressor is confronted, the individual becomes catatonic. B. stressful situations do not illicit psychological responses in individuals, only physiological responses. C. an individual will experience consistent physiological reactions to stress regardless of the stressor. D. stress is seen as invasive and negative, thus it cannot have a positive impact due to its depressive nature. Stress is the body's response to stimuli that are perceived as threatening or challenging. As proposed by Hans Selye, and supported by his research, the physiological response to stress is universal (choice C is correct). When a stressor is confronted, a catatonic state is somewhat unlikely; furthermore, this condition is considered to be a psychophysiological symptom of schizophrenia (choice A is wrong). When presented with stressful stimuli, the General Adaptation Syndrome suggests that individuals immediately respond by activating the sympathetic nervous symptom, the fight-or-flight response, and respond indirectly by psychological adaptation (choice B is wrong). Stress is not always negative. Eustress is a positive stress and can serve as motivation and even improve performance on tasks (choice D is wrong).

(Class 5: Passage 22) Stress is the body's response to stimuli that are perceived as threatening or challenging. As proposed by Hans Selye, and supported by his research, the physiological response to stress is universal (choice C is correct). When a stressor is confronted, a catatonic state is somewhat unlikely; furthermore, this condition is considered to be a psychophysiological symptom of schizophrenia (choice A is wrong). When presented with stressful stimuli, the General Adaptation Syndrome suggests that individuals immediately respond by activating the sympathetic nervous symptom, the fight-or-flight response, and respond indirectly by psychological adaptation (choice B is wrong). Stress is not always negative. Eustress is a positive stress and can serve as motivation and even improve performance on tasks (choice D is wrong).

What part of the brain plays a large role in experiencing and interpreting emotion? A. Occipital lobe B. Corpus callosum C. Limbic system D. Cerebellum

(Class 5: Passage 22) The limbic system is a system of connected structures buried deep within the cerebrum, including the hippocampus, the amygdala the olfactory bulbs and the limbic lobe, and is where most emotional experience and processing takes place (choice C is correct). The occipital lobe of the brain is in the anterior part of the cerebrum and is associated with perception and recognition of auditory stimuli, memory, and speech; this lobe is not associated with experiencing and interpreting emotion (choice A is wrong). The corpus callosum is the white matter of the brain (composed of many myelinated axonal tracts and projections) that connects the two hemispheres of the brain; this area is not directly associated with experiencing and interpreting emotion (choice B is wrong). The cerebellum is associated with regulation and coordination of movement, posture, and balance; it is not associated with experiencing and interpreting emotion (choice D is wrong).

What type of reinforcement schedule is applied if a response is rewarded after an unpredictable amount of time has passed, and what is predicted about the response to this schedule? A. Variable ratio; this schedule creates a high, steady rate of response. B. Variable ratio; this schedule produces a slow, steady rate of response. C. Variable interval; this schedule produces a slow, steady rate of response. D. Variable interval; this schedule creates a high, steady rate of response. A variable interval reinforcement schedule describes when a response is rewarded after an unpredictable amount of time has passed (choices A and B can be eliminated). This schedule produces a slow, steady rate of response (choice D can be eliminated and choice C is correct). An example of this might be checking your email (which often occurs at variable time intervals). A variable ratio reinforcement schedule describes when a response is reinforced after an unpredictable number of responses. This schedule creates a high steady rate of responding. Gambling and lottery games are good examples of a reward based on a variable ratio schedule.

(Class 6: FSQ Neuroscience, Memory, and Learning) A variable interval reinforcement schedule describes when a response is rewarded after an unpredictable amount of time has passed (choices A and B can be eliminated). This schedule produces a slow, steady rate of response (choice D can be eliminated and choice C is correct). An example of this might be checking your email (which often occurs at variable time intervals). A variable ratio reinforcement schedule describes when a response is reinforced after an unpredictable number of responses. This schedule creates a high steady rate of responding. Gambling and lottery games are good examples of a reward based on a variable ratio schedule.

You are training a dog to perform a trick on command, using treats as a reward. According to operant conditioning principles, what type of reinforcement schedule is most ideal for the acquisition phase of training and the maintenance phase of training, respectively? A. Continuous; variable ratio B. Variable interval; continuous C. Continuous; variable interval D. Variable ratio; continuous

(Class 6: FSQ Neuroscience, Memory, and Learning) According to operant conditioning principles, a continuous reinforcement schedule is most ideal for the acquisition phase of training and a variable ratio is most ideal for the maintenance phase of training, respectively. In continuous reinforcement, the desired behavior is reinforced every time it occurs; this schedule is best used during the initial stages of training a dog to perform a behavior because it helps create a strong association between the behavior and the reward (choices B and D can be eliminated). Once the response is firmly acquired, a variable ratio schedule, where a treat is given after a random number of correct responses, is the schedule that is best for maintaining that behavior. Variable interval schedules reward the desired behavior after an unpredictable amount of time has passed. This schedule produces a slow, steady rate of response, but is not as good as a variable ratio schedule for maintenance of a learned behavior (choice C can be eliminated and choice A is correct).

In the classic Little Albert experiment, researchers conditioned a nine-month-old boy to fear a white rat. The researchers observed that the boy also demonstrated fear in response to similar stimuli including a white dog, a white rabbit, a white fur coat, and even a white Santa Claus beard. This fear response to similar objects is known as: A. response discrimination. B. stimulus discrimination. C. stimulus generalization. D. response generalization.

(Class 6: FSQ Neuroscience, Memory, and Learning) In conditioning, stimulus generalization is the tendency for the conditioned stimulus (white rat) to evoke similar responses in the subject (Little Albert) after the response has been conditioned; in this case, Little Albert is demonstrating stimulus generalization because he is producing the conditioned response (fear) to lots of different but similar white, fluffy stimuli. Response generalization occurs when an organism emits a different response to a conditioned stimulus; so if instead of fear, Little Albert demonstrated anger in response to the white rat, this might be an example of response generalization (choice D is wrong). In classical conditioning, discrimination is the ability to differentiate between a conditioned stimulus and other stimuli that have not been paired with an unconditioned stimulus; therefore stimulus discrimination would mean that Little Albert responds only to the white rat (choice B is wrong) and response discrimination means that he responds only with fear (choice A is wrong).

A researcher designs a controlled laboratory experiment to measure changes in human brain activity in response to various visual and auditory stimuli. Which of the following scans would be LEAST appropriate to use? A. PET B. fMRI C. CT D. EEG

(Class 6: FSQ Neuroscience, Memory, and Learning) Make sure to highlight the word LEAST. PET (positron emission tomography), fMRI(functional magnetic resonance imaging), and EEG (electroencephalogram) all provide images of functional aspects of the brain (i.e., areas that are active will light up on the screen). Accordingly, these tests can measure brain activity changes that occur in response to stimuli (choices A, B, and D would be appropriate scans and can be eliminated). In contrast, a CT (computerized tomography) scan simply provides images of structural aspects of the brain. This type of scan would not be appropriate for measuring changes in brain activity in response to various stimuli (choice C is not appropriate and is the correct answer choice).

A common trick for remembering a list of items is to envision those items on different parts of your body. For example, to memorize a grocery list of milk, apples, celery, carrots, etc., you could envision a gallon of milk balanced on your head, an apple in your mouth, celery and carrots coming out of each ear, and so on. This memory trick is known as: I.) the dual-encoding hypothesis. II.) the method of loci. III.) chunking. A. III only B. II only C. I and II only D. I only

(Class 6: FSQ Neuroscience, Memory, and Learning) Since both Items I and II appear in exactly two answer choices, and Item II is shorter, start evaluating it. Item II is true: the method of loci involves imagining something (like your body) and leaving a visual representation of a word to be remembered at different parts of your body (choices A and D can be eliminated). Since neither of the remaining choices includes Item III, it must be false and you can focus on Item I. Item I is true: the dual coding hypothesis suggests that it is easier to remember words with associated images than either words or images alone. Thus, by encoding both a visual mental representation of an associated word, there are more connections made to the memory and an opportunity to process the information at a deeper level (choice B can be eliminated and choice C is correct). Note that Item III is false: chunking involves organizing the items to be remembered into chunks or groups, which is not being described here.

Emotions have been located in multiple structures within the brain. Which of the following areas is most closely associated with emotions? A. Parietal lobe B. Corpus callosum C. Limbic system D. Medulla

(Class 6: FSQ Neuroscience, Memory, and Learning) The limbic system is comprised of numerous structures of the brain and is closely linked to emotion. The limbic system itself is also involved in other processes including olfaction, attention and memory. Within the limbic system, the major emotional structure is the amygdala, which appears to be responsible for tagging memories with emotional significance. The limbic system is closely connected to the prefrontal cortex, which also serves to process cognitive experiences of emotion. The parietal lobe is found on the top of the brain and is largely responsible for sensation and language (choice A is wrong). The corpus callosum is the thick bundle of neurons that connects the left and right hemisphere. It allows the two sides of the brain to communicate and is not specifically involved in emotional processing (choice B is wrong). The medulla is a brain stem structure that is primarily responsible for autonomic processes such as breathing and regulating the heartbeat.

Which of the following brain structures is thought to regulate executive functions, including behavioral inhibition, planning, and decision-making? A. Prefrontal cortex B. Limbic system C. Reticular activating system D. Thalamus

(Class 6: FSQ Neuroscience, Memory, and Learning) The prefrontal cortex plays a role in executive functions; these are higher order thinking processes such as planning, organizing, inhibiting behavior, and decision making. Damage to this area may lead to inappropriateness, impulsivity, and trouble with initiation. This area is not fully developed in humans until they reach their mid-twenties, explaining the sometimes erratic and emotionally charged behavior of teenagers. The limbic system plays an important role in emotion, motivation, and memory (choice B is wrong). The reticular activating system is responsible for arousal and sleep-wake cycles (choice C is wrong). The thalamus plays an important role is relaying sensory and motor information, and also plays an important role in regulating consciousness and alertness (choice D is wrong).

Which of the following is the best example of insight learning? A. A grade school student receives a sticker for each homework assignment she completes. B. A teenager always takes the bus to his mom's workplace after school. When he gets his license, he drives for the first time to her workplace, and even though he never explicitly paid attention to the bus route, is easily able to navigate. C. A toy falls behind a bookshelf. A child tries to reach for it but can't get to it. He gets a stick and tries to pull the toy toward him without success. Frustrated he stops for a while, and then suddenly realizes that if he pushes the toy to the other side of the bookshelf, he can easily reach it from that side. He is then successful at obtaining the toy. D. A young girl has never poured her own milk and cereal but has watched her mom do it countless times. One morning her mom is still upstairs and she pours her own cereal and milk.

(Class 6: FSQ Neuroscience, Memory, and Learning) This is the best example of insight learning. Insight learning involves the abrupt realization of a problem's solution, which requires the ability to visualize the problem and the solution internally, before initiating a behavioral response. In the example with the boy and his toy behind a bookshelf, he has a flash of insight that helps him to figure out how to fetch the toy, which is the best example of insight learning. Receiving a sticker for completed homework assignments demonstrates positive reinforcement, an operant conditioning principle (choice A is wrong). A teenager who knows how to navigate a familiar route that he has never actually paid attention to is demonstrating latent learning (choice B is wrong). The girl who pours her own milk and cereal is modeling behavior she has seen her mom perform, so this is best described as vicarious learning (choice D is wrong).

Groupthink

-Group needs to reach a consensus -Decision-making can be more irrational and dysfunctional More likely to occur when: -any time there's a decision -the group is overly optimistic and strongly believes in its stance. (they believe they're right) -the group justifies its own decisions and demonizes those of its opponents. -dissenting opinions, information, and facts are prevented from permeating the group (a process called mindguarding) (group doesn't let other opposing info come in) -individuals feel pressured to censor their own opinions in favor of the perceived 'consensus', which creates an illusion of group unanimity.

Social loafing

-Group working together to complete a task -Individuals put in less effort than they would if they're working alone

After suffering a brain injury, a man is no longer able to remember facts and personal information he learned in the past, but can still form new memories. Furthermore, he remembers how to perform learned motor functions, like ride a bike and drive a car. Which of the following most accurately describes his memory impairment? A. He is likely suffering from anterograde amnesia for explicit memory, but his implicit memory appears intact. B. He is likely suffering from retrograde amnesia for explicit memory, but his implicit memory appears intact. C. He is likely suffering from anterograde amnesia for implicit memory, but his explicit memory appears intact. D. He is likely suffering from retrograde amnesia for implicit memory, but his explicit memory appears intact.

(Class 6: FSQ Neuroscience, Memory, and Learning) This patient is likely suffering from retrograde amnesia, which describes the loss of memory from before the injury (choices A and C can be eliminated). Also, the retrograde amnesia is specific for explicit memory, which includes facts and personal information) (choice D can be eliminated and choice B is correct). Note that because he is still able to perform learned motor functions his implicit (or procedural) memory appears intact. Anterograde amnesia describes when memory can no longer be consolidated from the point of injury forward, which is not occurring in this scenario.

Ben has been diagnosed with Post-Traumatic Stress Disorder. He developed his disorder by experiencing a trauma of being attacked by a wild animal while taking a hike in a state park. During the attack, a strong scent of pine was emanating from the pine trees in the immediate vicinity. A few months later, Ben was walking to the store on Main St. and noticed some pine trees lining the sidewalk. When he smelled the pine, he began feeling panicky and started having flashbacks of his attack. Now he avoids that grocery store and walks to one that is even farther so he doesn't have to walk past those pine trees. Which of the following learning theories are evident in this scenario? I.) Classical conditioning II.) Operant conditioning III.) Observational learning A. I only B. II only C. I and II only D. I, II, and III Item I is true: Ben has experienced classical conditioning, with the smell of pine being associated with the fear of the attack, as evidenced by his feelings of panic and autonomic arousal at the smell of the pine (choice B can be eliminated). Item II is true: Ben is also engaging in operant conditioning in the form of negative reinforcement. By avoiding the smell of the pine trees on his walk, he is effectively reducing his panic in the short term, therefore removing a negative stimulus from the environment and making it more likely that he will walk the longer distance to another grocery store (choice A can be eliminated). Item III is false: There is no evidence that Ben learned his behavior by imitating or emulating others, which would indicate observational learning (choice D can be eliminated). Therefore, both classical conditioning and operant conditioning are present in this scenario (choice C is correct).

(Class 6: Passage 16) Item I is true: Ben has experienced classical conditioning, with the smell of pine being associated with the fear of the attack, as evidenced by his feelings of panic and autonomic arousal at the smell of the pine (choice B can be eliminated). Item II is true: Ben is also engaging in operant conditioning in the form of negative reinforcement. By avoiding the smell of the pine trees on his walk, he is effectively reducing his panic in the short term, therefore removing a negative stimulus from the environment and making it more likely that he will walk the longer distance to another grocery store (choice A can be eliminated). Item III is false: There is no evidence that Ben learned his behavior by imitating or emulating others, which would indicate observational learning (choice D can be eliminated). Therefore, both classical conditioning and operant conditioning are present in this scenario (choice C is correct).

Operant conditioning is also important to psychopathology. An example of this is an individual with Social Anxiety Disorder avoiding going out with a new group of people. This avoidance means the person does not experience the anxiety of being in a new social situation which makes the behavior more likely occur in the future, to the detriment of the individual's social life and functioning. The behavior of the individual in the final paragraph who suffers from Social Anxiety Disorder can be explained with operant conditioning principles. This individual experiencing reduced anxiety by avoiding social situations can best be described as: A. positive reinforcement. B. negative reinforcement. C. positive punishment. D. positive punishment.

(Class 6: Passage 16) The behavior of the individual with Social Anxiety Disorder is described as avoiding a stressful situation, and that avoidance being more likely to occur in the future because they are removing a negative stimulus from their environment through avoidance. The type of conditioning that makes a behavior more likely to occur in the future is reinforcement, not punishment. Additionally, the removal of a negative stimulus represents negative reinforcement in this case (choice B is correct). The individual is not adding a positive stimulus to the environment to make the behavior more likely to occur, as is the case with positive reinforcement (choice A is wrong). Punishment is when the behavior is less likely to occur due to the consequences of the environment, which is not the type of operant conditioning taking place in this context (choices C and D are wrong).

Split brain patients are those who have had their corpus callosum, the white matter that connects the brain's two hemispheres, severed. As a result, when a split brain patient is shown a shape that is isolated to his left visual field only, he: A. will be not be able to consciously see it but will guess the name more often than chance. B. will not have any problems seeing or naming it. C. will be not be able to see it or name it. D. will be able to see it but not name it.

(Class 7: FSQ Sensation, Perception, Cognition, and Language) A split brain patient will be able to see an object in his left visual field (left nasal and right temporal) because the information from the temporal retina of the right eye will be processed in his right hemisphere. However, without a corpus callosum, this information cannot be transferred across to the left hemisphere where Broca's area is localized, so he will not be able to say what the shape is (choice D is correct, and choices B and C are wrong). "Blindsight", the ability to accurately "guess" what an object is without consciously seeing it, tends to occur in patients with primary visual cortex damage and would not be seen with a split brain patient as his visual cortext is intact (choice A is wrong).

Which theoretical perspective would propose that humans possess a "language acquisition device" that partially explains our ability to readily acquire language at an early age? A. Humanistic perspective B. Nativist perpsective C. Behavioralist perspective D. Interactionist perspective

(Class 7: FSQ Sensation, Perception, Cognition, and Language) Linguist Noam Chomsky, of the nativist perspective, proposed that humans are born hard-wired to learn language, and suggested this ability was due to the presence of a "language acquisition device" (LAD), an innate feature unique to the human mind that allows people to gain mastery of language from limited exposure during the sensitive developmental years in early childhood (choice B is correct). The humanist perspective (a la Carl Rogers) did not theorize about humans' ability to acquire language (choice A is wrong). Behavioralists (like B. F. Skinner), on the other hand, believed that humans learn language through a series of operant conditioning steps and stages, wherein young children are praised for making sounds that resemble words and eventually are conditioned through shaping to produce language (choice C is wrong). Lastly, the interactionist perspective also did not propose that humans have a language acquisition device, but that children become fluent in language by interacting with fluent adults (choice D is wrong).

Patient RK goes to the hospital weekly for dialysis. After seeing several news reports about MRSA outbreaks in hospitals around the country, RK becomes convinced that he will contract MRSA if he goes to the hospital for his weekly appointment, so he decides to skip it. This faulty logic is known as: A. representativeness heuristic. B. confirmation bias. C. mental set. D. availability heuristic.

(Class 7: FSQ Sensation, Perception, Cognition, and Language) Patient RK's faulty logic in this example is known as the availability heuristic. The availability heuristic is a mental shortcut that relies on immediate examples that come to mind. When RK is thinking about going to the hospital for his weekly dialysis, a number of examples of MRSA outbreaks in hospitals around the country immediately spring into his mind; as a result, he judges the likelihood of contracting MRSA as far more frequent and possible than it actually is (choice D is correct). The representativeness heuristic occurs when one estimates the likelihood of an event occurring by comparing it to an existing mental prototype; these prototypes are what each person thinks are the most relevant or typical example of a particular event or object (choice A is wrong). Confirmation bias is a tendency to search only for information that confirms our preconceived thinking, rather than information that might not support it (choice B is wrong). The tendency to only see solutions that have worked in the past is known as a mental set (choice C is wrong).

Santiago is chatting with his colleagues at a business luncheon, with the CEO seated at the other end of the table. Which of the following would be the best example of the cocktail party effect? A. Santiago pays close attention to what his colleagues are saying, but immediately notices when the CEO suddenly looks angry. B. Santiago pays close attention to what his colleagues are saying, but is equally attentive to everything the CEO says to those around him. C. Santiago pays close attention to what his colleagues are saying, but his attention is suddenly diverted when the CEO says the word "layoffs". D. Santiago pays close attention to what his colleagues are saying and is consequently unaware that the CEO has just mentioned Frank's name several times.

(Class 7: FSQ Sensation, Perception, Cognition, and Language) The cocktail party effect is the phenomenon by which we can filter out a conversation until we hear something salient, then switch our attention to that conversation. Treisman's model of attenuation adapted the Broadbent filter model to allow for detection of stimuli even when the individual is not attending to it. Santiago would likely switch his attention to the CEO's conversation upon hearing a personally significant word such as "layoffs" (choices B and D are wrong). The cocktail party effect only deals with auditory stimuli (choice A is wrong and choice C is correct).

The idea that language encodes cultural and cognitive properties that affect the way that people think, such that speakers of different languages will tend to think and behave differently depending on the language they use, is known as: A. the Linguistic Relativity hypothesis. B. the Gestalt theory. C. cognitive dissonance. D. Weber's Law.

(Class 7: FSQ Sensation, Perception, Cognition, and Language) The idea that language encodes cultural and cognitive properties that affect the way that people think, such that speakers of different languages will tend to think and behave differently depending on the language they use, is known as the Linguistic Relativity hypothesis. The central principle of gestalt psychology is that the mind forms a perceptual whole while simultaneously considering separate parts; this theory does not deal with the interaction between cognition and language (choice B is wrong). Cognitive dissonance asserts that we will strive to align our thoughts, beliefs, and behaviors in order to avoid internal tension, or dissonance; cognitive dissonance does not suggest that language impacts cognition as described (choice C is wrong). Weber's Law states that two stimuli must differ by a constant proportion (not a constant amount) in order for us to perceive the difference, and does not pertain to language and cognition (choice D is wrong).

Suppose that a teenage girl comes into the emergency room with a baby. She is very thin, and is acting strangely. The ER physician immediately assumes that she is a young, unwed mother on drugs. As it turns out, she is actually a middle-class honors student, who was taking care of her little brother when he suddenly became ill; her erratic behavior is because she is freaked out and doesn't know what to do. The physician's snap judgment about her can best be described by the: A. confirmation bias. B. availability heuristic. C. representativeness heuristic. D. mental set.

(Class 7: FSQ Sensation, Perception, Cognition, and Language) The physician's snap judgment about the teenage girl can best be described by the representativeness heuristic, one type of mental shortcut that is used when making quick decisions and judgments. In this example, the physician may estimate the likelihood that this teenager is an unwed mother and drug addict by comparing her appearance and behavior to an existing prototype of what an unwed teenage mother on drugs looks and acts like. The prototype is what the physician thinks is the most typical example of this type of person, perhaps arrived at by years of interacting with these patients. The confirmation bias occurs when we favor information that confirms our previously held beliefs and ignore or reinterpret information that contradicts those beliefs (choice A is wrong). The availability heuristic is a tendency to make judgments based on how readily available information is in our memories. If a memory is readily available, we may think the idea is more common than it actually is (choice B is wrong). The tendency to only see solutions that have worked in the past is known as a mental set; while in many cases our past experiences help us solve current issues, a mental set can also make it difficult to see novel or creative ways of approaching current problems (choice D is wrong).

A specific type of mental set that involves only being able to see solutions to a problem that uses objects in their normal or expected manner is known as: A. an algorithm. B. functional fixedness. C. availability heuristic. D. belief perseverance.

(Class 7: FSQ Sensation, Perception, Cognition, and Language) This is known as functional fixedness. With functional fixedness, there is a tendency to perceive the functions of objects as fixed and unchanging, which prevents us from using objects in novel or innovative ways to potentially solve problems. An algorithm is not a mental shortcut, but rather a step-by-step procedure for arriving at an answer or solution; algorithms ensure that the appropriate solution will be reached, but may be time-consuming and complex (choice A is wrong). The availability heuristic is a tendency to make judgments based on how readily available information is in our memories. If a memory is readily available, we may think the idea is more common than it actually is (choice C is wrong). Belief perseverance is the tendency to reject convincing proof and become even more tenaciously firm about an existing idea, even despite contradictory evidence (choice D is wrong).

You found a great new coffee shop to study at. It's very quiet and none of your classmates have discovered it yet, so you are able to study in peace. One day you take a short break from studying to listen to some music on headphones. You notice that you don't need to turn your volume up very loud because the surroundings are so quiet, but you are having a problem getting the volume at the right level. At level 2 the volume is too loud, but at level 1 it's too soft. How frustrating... this was never a problem when the volume was higher in loud environments! What principle best explains this phenomenon? A. Gestalt theory of psychology B. Background noise C. Signal detection theory D. Weber's Law

(Class 7: FSQ Sensation, Perception, Cognition, and Language) Weber's Law describes why differences in stimuli seem greater at lower levels than at higher levels. Although the difference between volume increments remains constant, the proportion of the overall noise level decreases as the volume increases. This makes the difference between the lower volume levels more perceptible (choice D is correct). The Gestalt theory of psychology describes a process by which incomplete but logically organized stimuli are "filled in" by the brain during perception; this theory does not explain why you perceive such a large gap between the sound volume on levels 1 and 2 (choice A is wrong). While there is a difference in background noise between the loud café and the quiet café, the noise level itself does not explain why it is difficult to get a comfortable volume setting while using headphones (choice B is wrong). Signal detection theory describes a way to separate meaningful signals from "noise"; this also does not explain the phenomenon described in the question stem (choice C is wrong).

How does the brain process visual input from watching a movie? A. Form and depth first, then movement, and finally color B. All elements are processed simultaneously C. Movement first, then depth, form and color D. First the form and color, then the movement

(Class 7: FSQ Sensation, Perception, Cognition, and Language) When watching a movie or other complex visual stimulus, all of the elements (form, depth, color, movement) are processed simultaneously. This process is referred to as "parallel processing". Under normal circumstances, the different aspects of visual information are not processed in a step-wise, or serial, fashion.

Which of the following is NOT true regarding stereotype threat? A. Telling a group of females that "females tend to perform more poorly on this math test than males" before the math test results in lower average scores for the females. B. White males are unaffected by stereotype threat because they are the majority group to which other groups are often compared. C. Emphasizing students' race before a standardized test often produces lower scores for ethnic and racial minorities. D. Females tend to score higher on math-based tests when testing in a group of all females (rather than a mixed group of females and males).

(MCAT Practice Test Course Test 3) Any group can be subject to stereotype threat, regardless of if the group is a majority or minority group. For example, when a group of white male students, who scored high in math on the SAT, were told that they were taking a test to "determine why Asian students typically outperform other students in math," they performed much worse in math. Stereotype threat can cause anyone to perform worse when they believe the negative stereotype could be true or worry about confirming a stereotype about a group to which they belong (choice B is correct). Numerous studies have shown that emphasizing the stereotype that males are better at math than females will produce lower math performance for females; this is one of the classic stereotype threat examples (choice A is wrong). Similarly, emphasizing an individual's race will also often lead to poorer academic performance, since the stereotype for most racial and ethnic minorities is that they do not perform as well on standardized tests as white people do; this is also a classic stereotype threat example (choice C is wrong). When females perform math-based assessments in all-female groups, stereotype threat is not activated, because they are less aware of their female-ness while taking the exam, so they do tend to perform much better; this is also a classic stereotype threat example (choice D is wrong).

The idea that foreign-born students might be negatively affected by native-born students' cultural norms and values can best be explained by: A. cultural relativism. B. prejudice. C. ethnocentrism. D. discrimination.

(MCAT Practice Test Course Test 3) Ethnocentrism is the tendency to believe that one's own ethnic or cultural group is the most important one, and that all other groups are measured against one's own. Ethnocentric thinking typically involves making false or negative assumptions about a minority ethnic or cultural group (choice C is correct). Cultural relativism is seen as the opposite of ethnocentrism, as cultural relativism is the principle of viewing aspects of a different culture from the viewpoint of that culture, rather than from one's own viewpoint (choice A is wrong). Prejudice is a negative and often unjustified attitude or stereotypical belief about an individual or a group, and it is not specific to race or culture (choice B is wrong). Discrimination is actual behavior, typically negative, towards an individual or group (choice D is wrong).

A person is presented with four unlabeled boxes that are 8, 9, 11, and 12 pounds each and one 10-pound box that is labeled. He first picks up the 10-pound box, followed by the 9-pound box, and says that they are the same weight. He then picks up the 10-pound box, followed by the 11-pound box, and says that they are the same weight as well. However, when he compares the 8 and 10-pound boxes, he notices that the 8-pound box is lighter, and when he compares the 12 and 10-pound boxes, he notices that the 12-pound box is heavier. Which of the following is true in this scenario? A. The absolute threshold is 10 pounds B. The difference threshold is 2 pounds C. The difference threshold is 4 pounds D. The just noticeable difference is 4 pounds

(MCAT Practice Test Course Test 3) In this scenario, the 10-pound box is a standard stimulus and the others are comparison stimuli. The two-pound difference between 8 and 10 and between 10 and 12 that is required for the person to tell that the boxes are different weights is the difference threshold, or the amount of energy that needs to be added to, or subtracted from, the standard stimulus before there appears to be a difference (choice B is correct; choice C is wrong). An absolute threshold is the amount of stimulus energy needed for a person to perceive it at all, which is not discussed in this scenario (choice A is wrong). The just noticeable difference (JND) is the same value as the difference threshold; in this case, two pounds would equal one JND, four pounds would equal two JNDs, etc. (choice D is wrong).

Symbolic Interactionism

-Individuals communicate with each other using culturally learned symbols. -Theorist was George Herbert Mead -People act towards things based on meaning. -Meanings are derived from social interactions and adapted through individual interpretation. -Symbols are culturally derived social objects that have shared meanings, which are created and maintained through social interaction. -Micro level theory -The central theme is that human life is lived in the symbolic domain. -Through language and communication, symbols provide the means by which reality is constructed. (e.g. red rose in US symbolized romance and love, but the red rose means nothing in other countries)

The vestibular sense detects motion and is involved in balance by way of what mechanism? A. Control of autonomic functions by the pons B. The endolymph in the cochlea contains hair cells that detect motion and rotation of the head C. The endolymph in the utricle, saccule, and ampullae contain hair cells that detect motion and the rotation of the head D. Coordination of motion in cerebellum

(MCAT Practice Test Course Test 3) The endolymph in the utricle, saccule, and ampullae—the semicircular canal of the inner ear—contain hair cells that detect motion and the rotation of the head. This aids in the sense of balance (choice C is correct). The pons in the hindbrain controls some autonomic functions and coordinates movement and thus is related to the sense of balance but that control of autonomic functions on its own is not the vestibular sense (choice A is wrong). The endolymph in the cochlea contains hair cells that are designed to transmit auditory vibrations for the sense of hearing, not proprioception or balance (choice B is wrong). The cerebellum coordinates complex movements and damage to that area results in difficulties in movement and balance, but again this is not the specific mechanism of the vestibular sense (choice D is wrong).

A father asks his underaged son if he has been smoking after noticing the smell of cigarettes on his son's clothes. The son admits to smoking for the very first time because his friends asked him to try it. His father lets him off with a warning but no punishment. According to labeling theory, this behavior is categorized as: A taboo. B identification. C primary deviance. D conformity.

(MCAT Practice Test: MedSchool Coach) According to labeling theory, behaviors are labeled as deviant based on what society judges as acceptable behavior. Primary and secondary deviance are labels given based on the consequences and social reaction to deviant behaviors. A child's first attempt at smoking is not deviant enough to receive serious consequences or a more permanent label like secondary deviance, and is categorized as primary deviance (choice C is correct). Taboos are norms society has deemed forbidden and immoral and are not part of labeling theory (choice A is incorrect). Identification is a type of conformity in which people act similarly to those they admire. Both conformity and identification are not part of labeling theory (choices B and D are incorrect).

Which of the following scenarios best describes compliance? A A child asks to go to the playground more often after his friends teach him how to ride the swings. B A man attends the neighborhood meeting on curfew regulations because his neighbor asks him to attend. C Two close friends notice they are wearing the same shirt and agree to match again next week. D A teenager completes his mandatory community service as ordered by a court judge.

(MCAT Practice Test: MedSchool Coach) Conformity refers to how we change our actions and/or thoughts to match the actions and thoughts of others. There are three types of conformity: compliance, internalization, and identification. Compliance is a type of conformity that refers to changing our actions to match the needs of another person and does not require a change in our thoughts. The man may not like neighborhood meetings, but may comply and attend because he was asked (choice B is correct). Internalization is a stronger type of conformity in which our own values are changed by the thoughts and behaviors of others. The child has internalized that swings are fun after learning from his friends (choice A is incorrect). Two friends unintentionally wearing the same shirt may be indicative of some conformity due to friendship but is not an explicit example of compliance. Furthermore, since there is no indication that one friend only agrees to match next week because the other suggests so, this is not an example of compliance (choice C is incorrect). Obedience is an idea separate from conformity in which people follow the orders of an authority figure. The teenager completes his mandatory community service because he must obey the order of the court judge, not because he agrees to comply and fulfill the community service (choice D is incorrect).

Ethnocentrism refers to: A judging another person's culture as better than one's own culture. B judging another person's culture by the standards of one's own culture. C judging a culture by its own cultural standards. D judging one's own culture by the standards of another person's culture.

(MCAT Practice Test: MedSchool Coach) Ethnocentrism is defined as the judgment of another person's culture by the standards of your own culture and can lead to the perception of one's own culture as superior to other cultures (choice B is correct). Xenocentrism is defined as judging one's own culture from the lens of another due to the belief that the other culture is superior to one's own (choices A and D are incorrect). Cultural relativism is the opposite of ethnocentrism and refers to the judgment of a culture by its own standards; it is associated with the idea that there can be many types of cultures, all of which can be valid (choice C is incorrect).

In the history of the United States, the large-scale deployment of high-quality highways in and around urban areas is most directly associated with which of the following phenomena? A Industrialization B Suburbanization C Gentrification D Globalization

(MCAT Practice Test: MedSchool Coach) In the roughly 20-year period after World War II (1945-1965), the United States underwent a process wherein population flowed from urban cores to suburbs. One of the factors driving this process of suburbanization was the development of high-quality highway systems to make it easier for people to commute from suburbs to jobs in urban areas (choice B is correct). Industrialization occurred earlier in Western Europe and the United States (in the 1800s) and is more closely associated with the railroad in terms of infrastructure developments (choice A is incorrect). Gentrification happened more recently and is associated with the re-influx of population (typically more affluent) into the urban core (choice C is incorrect). Globalization refers to a set of changes in the late 20th century that resulted in closer links among countries in terms of communication and transport infrastructure (choice D is incorrect).

Max Weber

-Considered a founder of modern sociology -Theorized conflict theory -Modern societies are becoming increasingly rational and bureaucratic. They asserted that: -society was moving away from church and instead towards rationalization • there could be more than one source of conflict, not just income. • several factors can moderate people's reaction to inequality, such as agreement with authority figures, high rates of social mobility, and low rates of class difference. • the defining transformation of Western cultures was increasing rationalization, societies' trend toward increasing efficiency and away from traditional religious standards of spirituality and morality. • rationalization also promotes the proliferation of highly impersonal bureaucracies, agencies of non-elected officials that administer the laws of society. They also believed that. • ideas and beliefs like from celebrities or historical figures can exert a very powerful effect on society. • people base their actions on their personal interpretation of the meaning of the world around them. -His ideas led to symbolic interaction

There are five recognized tastes: sweet, salty, bitter, sour, and umami. Recent research suggests there is also a sixth, oleogustus, the unique taste of fat. The lead researcher on the study said that since there aren't any words that exist for this taste, they were forced to make it up, and it was very difficult to figure out if people really view this as unique sensation. This best demonstrates: A. gestalt principles. B. the Sapir-Whorf hypothesis. C. the major limitations of sensory processing. D. monocular cues.

(Practic Test 4) This example suggests that it was difficult to describe and define things for which we don't have language; this is an example of the Sapir-Whorf hypothesis, which suggests that the structure of a language determines/influences the thought and behavior characteristic of the culture in which it is spoken. In other words, without an actual word to define the fatty taste, it was difficult to determine that it was in fact a unique taste (choice B is correct). Gestalt psychology emphasizes the mental processes that perceive the "whole" of objects or situations. This school of psychology has been particularly influential in describing tendencies of human visual perception. Gestalt principles do not describe how a lack of language con influence perception (choice A is wrong). This example describes a major limitation of language to define a sensation, not a limitation of sensory processing itself (choice C is wrong). Monocular cues, such as relative size and texture gradient (among others) are cues that can be perceived with only one eye that help us judge distance (depth perception). They have nothing to do with taste (choice D is wrong).

In the semantic network model, what determines the strength of a connection between a node and an association? A. How frequently and deeply connections are made B. How closely the node and association are explicitly related C. How many links there are between the node and the association D. How many state-dependent cues have been created between the node and the association

(Practice Test 4) According to the semantic network model, the strength of a connection between a node and an association is related to how frequently and how deeply the connections are made (choice A is correct). The explicit relationship between the node and the association is meaningless unless that connection is constantly reinforced (choice B is wrong). Separate links between the node and the association—suggesting a more circuitous route—are indicative of the spreading activation pattern (choice C is wrong). State-dependent cues, in which familiar locales are used to trigger memories, do not determine the strength of the connection between a node and an association unless, once again, they are used with great frequency (choice D is wrong).

Suppose that Sarah is kneading a piece of clay in her hands. Which of the following is NOT a strategy relevant to conservation? A. Sarah realizes that the clay is the same piece of clay, regardless of its shape. B. Sarah notices that she can shape the clay into a ball, then a pancake, then back into a ball again. C. Sarah knows that the clay still exists even when her mother puts it back into its container. D. Sarah discovers that squeezing the clay in the middle forces it to become bulkier on the sides.

(Practice Test 4) Conservation is the term that Piaget used to describe children's recognition of constancy and consistency in physical matter despite changes in shape or container. This awareness is characteristic of the stage of concrete operational thought, which is usually achieved between the ages of 6 or 7 and 11 or 12. Conservation is realized through three insights: identity, reversibility, and reciprocity. Identity involves the child's recognition that the physical object is the same object, regardless of how it is manipulated (choice A is a strategy relevant to conservation and is therefore wrong). Reversibility involves the child's recognition that a given manipulation can be reversed to give the object its initial appearance (choice B is a strategy relevant to conservation and is therefore wrong). Finally, reciprocity involves the child's realization that a manipulation of one dimension or aspect yields a corresponding change in another aspect. As a result, the manipulation is understood to change the original object rather than create a new one (choice D is a strategy relevant to conservation and is therefore wrong). The realization that an object continues to exist despite its disappearance from the visual field is known as object permanence (choice C is not a strategy relevant to conservation and is therefore correct).

There are multiple sociological perspectives on deviance. Which of the theorists below are expected to consider the associated questions about non-normative behaviors? Differential association theorists; "How can people resist deviance?" Labeling theorists; "Who defines deviance?" Structural strain theorists; "How do norms affect deviance?" A. III only B. I and II only C. II and III only D. I, II, and III

(Practice Test 4) Item I is false: differential association theorists argue that deviant behaviors are learned through interactions between individuals and their communities. These theorists contend that people become deviant when they are exposed to deviant behavior and attitudes. "How can people resist deviance?" is therefore not a pertinent question (choices B and D can be eliminated). The main criticism of differential association is that it essentially reduces individuals to their environments. Because it states that those in deviant communities learn to be deviant themselves, the possibility of resistance is not contemplated. Item II is true: labeling theory asserts that behaviors are seen as deviant as the result of social processes of labeling. "Who defines deviance" is therefore a pertinent question (choice A can be eliminated). Labeling theorists do indeed address this concern; for example, these theorists are interested in the mechanisms through which power contributes to deviance labeling (e.g., agents of social control). Item III is true: structural strain theory suggests that deviant behaviors are the result of tension between the accepted social goals and the institutionalized means available to achieve those goals. "How do norms affect deviance?" is therefore a pertinent question (choice C is correct). Structural strain theorists consider the effects that social norms have on behavior, normative or not, and suggest that there is pressure to use deviant methods when the social structure does not support the achievement of the accepted goals.

How might a theorist in the tradition of symbolic interactionism explain deviance? A. Deviant behavior is a response to being labeled a deviant by others. B. Deviant behavior results from performing on the back stage when one should be performing on the front stage. C. Deviance is a failure to develop a looking-glass self. D. Deviance is a failure to resolve the dilemma of identity versus role confusion in adolescence.

(Practice Test 4) Labeling theory is a fundamental concept of symbolic interactionism. It states that deviance is not inherent in the act itself, but is rather an attribution from others (such as police, family, or clergy; choice A is correct). Back stage and front stage dynamics are an aspect of the dramaturgical approach. If one acts on the back stage when he or she should be performing on the front stage, the audience might experience more than it otherwise would (e.g., more details about a person's motivations or inner thoughts), but this would not necessarily be a deviant act (choice B is wrong). A looking-glass self describes how people shape their self-concepts based on how they think others perceive them (choice C is wrong). The dilemma of identity versus role confusion is one of Erikson's eight developmental stages; it is not an aspect of symbolic interactionism (choice D is wrong).

One advantage of using a longitudinal study design, such as the one described in the passage, is: A. the ability to conduct in-depth analyses of a few participants to get very rich and detailed data samples. B. the opportunity to follow a cross-section of participants of different types at the same time. C. the opportunity to follow one group of participants over an extended period and track their changes. D. the ability to see how many participants remain in the study and how many people withdraw.

(Practice Test 4) Longitudinal studies in developmental psychology research, such as the one in the passage, allow researchers to follow one group of participants over time. In this study, they were able to follow participants over a six-year period and track their changes (choice C is correct). Case study design (not a longitudinal study design) allows researchers the opportunity to conduct in-depth analyses of a few select participants, then emerge with very detailed data (choice A is wrong). Cross-sectional research (not a longitudinal study design) gives researchers a variety of participants of different types (e.g., different ages) that they can track at the same time (choice B is wrong). A drawback (not an advantage) of longitudinal research is that sometimes participants will withdraw from the research over the period of the study for a variety of reasons, including moving away, lack of interest, or even illness or death (choice D is wrong).

Research has also considered the salaries of medical professionals and the effects of compensation on overall life satisfaction. Reports of average earnings range from $156,000 to $315,000, with pediatricians reporting the lowest salaries and orthopedic surgeons and radiologists reporting the highest salaries. Based on this information, models of social stratification in the United States would consider the average anesthesiologist to be a member of which social class? A. Upper class B. Middle class C. Working class D. Lower class

(Practice Test 4) Most sociologists consider socioeconomic status (SES) when discussing social stratification. This measure is often defined in terms of power, prestige, and wealth owing to the common interrelationship among these factors. The reported salaries of specialists range from $157,000 to $316,000 for the lowest and highest earners listed, respectively. The average anesthesiologist is then expected to earn an intermediate amount (the research reports an annual average of approximately $310,000). While estimates differ, this earning potential alone suggests that anesthesiologists are middle class, perhaps upper middle class (choice B is correct). Middle class annual incomes range from $46,000 to the minimum earnings representing the upper class (this definition varies); 40% of the population is considered middle class. Despite the high earnings of these medical specialists in comparison to the national average, it is a common misconception that these professionals are "upper class". According to most models of social stratification, the income range for upper class is much higher. Upper class annual incomes range from millions to billions of dollars; a mere 3% of the population is considered upper class (choice A is wrong). There are medical professionals who earn comparable amounts when accounting for additional income and wealth (e.g., famous doctors who host their own medically-themed television shows can earn several million dollars each year). However, the information provided considers the average anesthesiologist. Working class annual incomes range from $19,000 to $45,000; 30% of the population is considered working class (choice C is wrong). Lower class annual incomes range from no income to $18,000; 27% of the population is considered lower class (choice D is wrong).

All of the following biological developmental milestones that change the brain occur during the adolescent years EXCEPT: A. Cell proliferation (particularly in the limbic system and prefrontal lobes) B. Myelination C. Synaptic pruning (of unused or unnecessary connections) D. Codification of neural networks for basic motor skills

(Practice Test 4) Neural network development for motor skills is part of early brain development and is essentially complete for basic motor skills (such as crawling, walking, jumping, and writing) before adolescence (choice D does not occur during adolescence and is therefore correct). Cell proliferation, particularly in the prefrontal lobes and in the limbic system, is one of the changes to the brain that occurs during adolescence, as the prefrontal cortex is required for abstract thinking (choice A is wrong). Myelination, the creation of myelin sheaths around neurons in the brain to strengthen connections, is another change that occurs during adolescence (choice B is wrong). Synaptic pruning, which clears the brain of unnecessary connections, is the third major developmental change in the brain during adolescence (choice C is wrong).

Which of the following describes how the brain uses parallel processing to process a visual stimulus? A. A different area of the brain is activated by each type of image (images, letters, etc.). B. Visual areas V1-V5 are utilized to analyze different aspects of an image simultaneously. C. Images are simultaneously processed using both top-down and bottom-up processing. D. Visual areas V1-V5 are utilized to analyze each aspect of an image in succession.

(Practice Test 4) Parallel processing describes the simultaneously processing of different aspects of a stimulus (choice B is correct), instead of analyzing the pieces of a stimulus step-by-step (choice D can be eliminated). Choice A is the definition of feature detection theory, and can be eliminated. Top-down and bottom-up processing are elements of gestalt psychology; the simultaneous use of both top-down and bottom-up processing is characteristic of most sensory processing, but is not the definition of parallel processing (choice C can be eliminated).

If a bisexual female reports feeling social pressure to be either lesbian or straight, which causes her significant stress, what term best describes the stress this subject is feeling about her social role? A. Role conflict B. Master status C. Role exit D. Role strain

(Practice Test 4) Role strain is when having a single status results in conflicting expectations; if a bisexual female reports feeling social pressure to be either lesbian or straight, which causes her significant stress, she is experiencing conflicting expectations about her bisexual identity (choice D is correct). Role conflict happens when there are conflicting societal expectations for multiple statuses held by the same person. For example, a male kindergarten teacher—societal expectations for being a man and being a kindergarten teacher easily come into conflict (choice A is wrong). Master status is when one part of someone's identity comes to dominate their identity, to the exclusion of other parts of their identity (choice B is wrong). Role exit is the process of disengaging from one role in order to take up another. An example is the process of disengaging from a "pre-med" identity to take on a "medical student" identity (choice C is wrong).

A secure attachment style is best demonstrated by which of the following scenarios? A. A five-year-old child who remains by her mother's side at a children's birthday party, not interacting with the other children B. A four-year-old child who runs toward her mother with open arms upon her return from work, but before reaching her turns around and instead runs away frightened. C. A four-year-old who confidently joins in a wide array of activities while at a birthday party and later seeks his mother's help, and is soothed by her, when a game of tag results in a skinned knee D. A five-year-old child who explores different parts of a playground, pushing his climbing of the jungle gym to dangerous heights, with the knowledge that his mother will not be paying attention

(Practice Test 4) Secure attachment is promoted by a responsive and available parent and is characterized by a child who is able to explore his or her environment secure in the knowledge that the parent will be there when needed. A child confidently exploring and joining in different activities at a birthday party demonstrates a secure attachment style. The child's act of seeking his mother's aid during a stressful time, such as when being injured, and the parent being available to comfort and soothe him, further displays secure attachment (choice C is correct). Reluctance to explore and instead clinging and remaining by the mother's side is indicative of an anxious-resistant insecure attachment style, in which the child is uncertain about the parent's availability because of previous separations. Because of this fear of being abandoned, he or she stays close and clings to the mother (choice A is wrong). Odd or awkward behaviors when separating or reuniting with the parent, such as extreme shifts between proximity-seeking and avoidance, are indicative of a disorganized attachment style. A child who, upon being reunited with her mother, initially runs toward her with open arms and then quickly runs away instead illustrates an extreme shift between proximity-seeking and avoidance. Disorganized attachment is usually the result of the child experiencing trauma, such as being physically, emotionally, or sexually abused by the parent. In this instance, the child sees the parent as both frightening and the only source of comfort, which causes great confusion and results in the eratic proximity-seeking and avoidance behavior (choice B is wrong). An anxious-avoidant attachment style results from the child being constantly rejected and rebuffed by the parent. These children learn to be independent early on, depending only on themselves because they learn that they cannot rely on the parent who is never available. A child exploring and climbing the jungle gym to dangerous heights displays this early independence. The child's knowledge about the lack of availability and responsiveness of the parent further provides evidence of an anxious-avoidant attachment style (choice D is wrong).

The myelin sheath is a dielectric that surrounds axonal membranes and facilitates electrical transmissions of neuronal signals. Demyelination is implicated in the exacerbation of Alzheimer's symptoms. Impairment of cognitive abilities due to demyelination most likely occurs because: A. increased speed of processing results in improved episodic memory. B. decreased speed of processing results in impaired semantic memory. C. decreased speed of processing results in improved episodic memory. D. decreased speed of processing results in impaired semantic memory.

(Practice Test 4) Semantic memory refers to the memory of meanings and other concept-based knowledge. As demyelination occurs, neuronal signals that carry meanings slow down, damaging the memory of them (choice D is correct). Episodic memory is the ability to contextualize the memory of autobiographical events; increased speed of neuronal processing could result in improved episodic memory, but this would not impair cognitive abilities and is not what the question stem asks (choice A is wrong). Decreased speed of neuronal processing would not result in memory improvement, whether semantic or episodic (choices B and C are wrong).

Suppose Subject 405 returned home after the experiment and watched a television program about mammalian predators. Before a commercial break, the program's narrator explained that bears secrete kairomones and challenged viewers to think of the primary chemical contained in bear kairomone. During the break subject 405 confidently told her roommate that "no doubt the primary chemical is sulfur!" Assuming that subject 405 has no familiarity with kairomones other than through her experience as a study participant, her assertion exemplifies which of the following? A. The availability heuristic B. The optimistic bias C. The representativeness heuristic D. The central route to persuasion

(Practice Test 4) The availability heuristic refers to people's tendency to solve problems or estimate probabilities based on information that is most readily available, or accessible, to their minds. Thus, Subject 405's assertion reflects her recent experience as a participant in the study rather than a logical approach toward problem solving (choice A is correct). The optimistic bias is people's tendency to assess the probability of personal success in future endeavors with more optimism than is warranted by actual facts (choice B is wrong). The representativeness heuristic refers to a problem-solving strategy by which people make choices based on their perception of which option is most characteristic of the issue in question. Since the question stem assumes that Subject 405 has no familiarity with kairomones other than through her participation in the study, there is no basis for Subject 405 to consider sulfur to be more representative of mammalian kairomones than any other substance (choice C is wrong). When people use logic or information-based arguments to persuade others, they are considered to be using the central route of persuasion. Subject 405, however, did not offer any such arguments to substantiate her claim (choice D is wrong).

Which one of the following regions of the brain is most likely to be implicated in the onset and degenerative progression of Alzheimer's disease? A. Hippocampus B. Thalamus C. Posterior occipital lobe D. Superior parietal lobe

(Practice Test 4) The hippocampus is associated with the formation, storage, and retrieval of memories. Since Alzheimer's often profoundly affects a sufferer's ability to remember, this region is most likely to be implicated in the onset and degenerative progression of AD (choice A is correct). The thalamus is a relay center for sensory information that is important for sensation and the perception of external stimuli. Since the primary symptom of Alzheimer's is memory loss, this is not the brain region most likely to be implicated (choice B is wrong). The posterior occipital lobe is critical to vision and the organization of visual information (choice C is wrong). The superior parietal lobe has many functions, including integration of spatial information and maintenance of internal representations; however, none of these functions involves the primary symptoms of AD (choice D is wrong).

The hippocampus is a subpart of which brain structure? A. Executive functioning B. Basal ganglia C. Occipital lobe D. Limbic system

(Practice Test 4) The hippocampus is part of the limbic system of the brain; a system which is responsible for many aspects of human functioning, including emotions and memory (choice D is correct). Executive functioning is generally thought to be regulated by the frontal lobes, which control inhibition and attention (choice A is wrong). The basal ganglia is situated at the base of the forebrain and is mainly comprised of striatum, the globus pallidus, the substantia nigra, the nucleus accumbens, and the subthalamic nucleus. It is associated with voluntary movement, procedural learning, and emotions, but is not associated with episodic memory as the hippocampus is (choice B is wrong). The occipital lobe of the brain is at the back of the head, and is primarily associated with vision (choice C is wrong).

Because researchers chose to use a questionnaire, which of the following was missing from this study? A. Dependent variables B. Scientific method C. Independent variables D. A control group

(Practice Test 4) The use of a questionnaire means that the researchers did not utilize a control group and thus did not have two groups participating simultaneously under different circumstances; this would simply not be possible with this methodology (choice D is correct). The dependent variables in this study are the preferences regarding age and gender of the students polled in the questionnaire—what the researchers were looking to study (choice A is wrong). The scientific method, in which hypotheses are empirically tested through the careful and systematic collection of relevant data. was used in this study and is ideally used in all scientific research (choice B is wrong). The independent variable here is the group of students polled (those between the ages of 18 and 30, identifying as either male or female; choice C is wrong).

A forty-year-old woman is admitted to the emergency room after a serious car accident. Hospital staff attempt to obtain the woman's emergency contact information in order to have someone come to the hospital as a source of support. The woman refuses to provide any contact information, stating that she does not need, want, or have anyone whom she wishes to be contacted in her behalf. The woman further adds that she prefers to live a solitary life that is not dependent on others. Based on this limited information, which of the following best describes her interactions with her primary caregivers during childhood and her resulting attachment style? A. A secure attachment style resulting from interactions with loving and supportive parents who were available and comforting when needed B. A disorganized attachment style resulting from interactions with parents who were sometimes positive and, at other times, harsh and rejecting C. An anxious-resistant insecure attachment style resulting from interactions between parent and child that left the child feeling uncertain about the availability of the parent during times of need D. An anxious-avoidant insecure attachment style resulting from rejection of the child by the parent during times of need

(Practice Test 4) The woman's reluctance to form attachments to others and preference for a solitary life may indicate that she formed an anxious-avoidant insecure attachment style during childhood. This style is the result of the primary caregiver constantly rejecting or rebuffing the child when he or she sought comfort or protection. Because of these repeated rejections, the child comes to expect the same from other people. At the extreme, these individuals prefer to live a life free of any attachments to others, much like the woman described above (choice D is correct). If the woman had established a secure attachment style during childhood, she would likely have numerous individuals with whom she has bonded and formed strong relationships. Thus, she would have individuals in her life from whom she would like to receive comfort and support (choice A is wrong). This woman does appear to have difficulty trusting others, which could be indicative of a disorganized attachment. However, disorganized attachment usually does not lead to disavowal of relationships and significant connections to others. Rather, the quest to form interpersonal connections is hampered by poor social skills and an inability to regulate emotions (choice B is wrong). An anxious-resistant insecure attachment style usually results from parents who are sometimes supportive and comforting, and at other times absent and unavailable, leaving the child uncertain about the parent's availability. This leads to preoccupation in adulthood with being abandoned by significant others and to hesitation in forming relationships; when these individuals do form relationships they may display clingy behaviors. With this attachment style, the woman in question would likely be very motivated to have significant people in her life available during such a time of need (choice C is wrong).

Karl Marx

-Considered a founder of modern sociology -Theorized conflict theory They asserted that: -capitalism is an economic system that encourages private ownership in order to produce profit and thereby wealth. -under capitalism, those who are able to use private ownership to produce profit are incentivized to minimize the wages of their workers, creating a cycle of greater and greater inequality. -workers would inevitably realize that they are being exploited, resulting in a series of workers' revolutions. -capitalism would then be replaced with socialism, an economic system in which most means of production are collectively owned in order to benefit all members of society equally (in theory).

mesolimbic pathway

-(Blueprint) The mesolimbic pathway, or reward pathway, is a pathway in the brain connecting the midbrain to the forebrain. It releases dopamine in response to rewarding stimuli, thus reinforcing behaviors that are perceived as pleasurable. -subcortical part of the brain involved in the processing of salience and reward -subcortical part of the brain involved in cognition and emotion -(Blueprint) The mesolimbic pathway (sometimes referred to as the mesolimbic reward pathway) is involved in addiction and compulsive behavior. This pathway, made up by the nucleus accumbens, the ventral tegmental area (VTA), and the area that connects them, the medial forebrain bundle, is one of the four dopaminergic pathways in the brain that are involved in motivation and emotional response. When activated by addictive drugs, it can give chemical, positive reinforcement for their continued usage. All activities that produce psychological dependence, including drug use, gambling, and even falling in love, activate the mesolimbic reward pathway.

socioeconomic status (SES)

-A measure of social class that is based on income, education, and occupation -A division of population based on occupation, income, and education. -Defines the economic and social position of a person in terms of income, wealth, education, and occupation

null hypothesis

-A prediction that there is no difference between groups or conditions, or a statement or an idea that can be falsified, or proved wrong. -the hypothesis that there is no significant difference between specified populations, any observed difference being due to sampling or experimental error

statistical significance

-A statistical statement of how likely it is that an obtained result occurred by chance -A p-value less than 0.05 (typically ≤ 0.05) is statistically significant.

Syllogistic reasoning

-A systematic process of logical thinking such that if A is related to B in X way, and if C is an example of A, then C will also be related to B in X way

Feminist theory

-A variety of perspectives on the treatment of women vs. men in society -Women deserve rights that are politically, socially, and economically equal to men. -Emerged from movements in the 19th and 20th centuries advocating for equal political, economic, and social rights of women in society. -Can be either a macro- or micro-level theory of society. -Can be divided into three "waves" or eras. -Around 1900, first-wave feminism focused on women's suffrage: the right to vote, to own property, to have equal rights within marriage, and to work for wages. -In the 1960s and 1970s, second-wave feminism focused on women's social liberties: gender equality, equal pay, sexual de-stigmatization, and reproductive rights. -Since the 1980s, third-wave feminism has focused on intersectionality, the study of how different social identities such as gender, race, class, etc. interact. -Glass escalator: refers to the way men are often fast tracked to advanced positions when entering primarily "pink collar" professions. -Glass ceiling: a metaphor representing an invisible barrier that prevents a given demographic from rising beyond a certain level in a hierarchy

Emile Durkheim

-Considered a founder of modern sociology -Theorist of functionalism They asserted that: -a dynamic equilibrium results in social structures working synergistically toward societal solidarity, resulting in a common consciousness that exerts a strong coercive influence on individual actors. -societies that have thrived for generations have evolved healthy self-regulating and self-correcting mechanisms. -dysfunctions in societies are caused by anomie (values of an individual that don't fit/align with values of society), which this person defined as a mismatch between the wider social standards and the standards of individuals or smaller groups within that wider society.

George Herbert Mead

-Considered a founder of social psychology -Theorized symbolic interactionism and "I" and "Me" -His ideas form foundation for symbolic interactionist school of thought. They asserted that the self develops in three stages: -Preparatory stage (0-2): Children learn to use language and symbols, but only mimic those around them. -Play Stage (2-6): Children learn to play the roles of others in pretend games such as "playing doctor". -Game Stage (7+): Children begin to understand the "generalized other", the generalized attitude of a large social group.

I and Me

-Individuals develop a social self, which he called the "me," and a response to that social self which he called the "I". -"Me" is set of behaviors and social self, "I" is sense of individuality (I am the funny and class clown in my class) -The "me" develops through interactions with others and consists of our interpretations of how the "generalized other" views us. -The "I" arises in response to the "me". The "I" is one's personal identity and individuality. -To Mead, the actual self consists of the balance of "I" and "me". -Thinking is simply an ongoing discussion between the "I" and "me".

Rational choice and exchange theory (exchange-rational theory)

-Individuals make decisions by comparing the costs and benefits -The goal is to maximize benefits and reduce costs -Borrowed from Economic Theory -Individuals respond to rewards and punishments -The goal is to maximize rewards and reduce punishments -Borrowed from Behavioral Psychology Sometimes known as Social Exchange theory Both are are usually micro-level theories.

Stigma

-It is defined as disapproval resulting from actual or perceived social deviance. -It can lead to demeaning labels that are applied to entire social groups. -Stigma is extreme disapproval of a person based on some behavior or quality of that person. Typically, a culture (or subculture) will stigmatize a person based on overt physical deformations (e.g., physical disability), deviant personal traits (e.g., drug addiction), or deviation from accepted norms of the group (e.g., being a "loose" woman in a traditional Latino subculture).

Conflict theory

-Parts of society work against each other in competition for limited resources. -Marx and Weber were theorists -Society is a competition for limited resources. -Individuals and groups compete for social, political, and material power. -Theorists are primarily concerned with imbalance in wealth, power, and prestige among major groups in society and how such imbalances competitively advantage the haves over the have-nots. -Macro-level theory.

Functionalism

-Parts of society work together to maintain stability (dynamic equilibrium). -Emile Durkheim was theorist -Cooperarion and contribution -Society is a complex system. -Individual parts work together to maintain solidarity, order, balance, and social stability. -compares society to a living organism. -Parts of a society (individuals, families, communities, groups, organizations, etc.) work together to maintain dynamic equilibrium. -Macro-level theory.

Social facilitation

-Performing a task in front of other people -Performance (1) improves for simple, well-ingrained tasks, and (2) gets worse for complex tasks

Peer pressure

-Presence of peers -Individuals feel pressured to conform to peer group (can be positive or negative)

Social constructionism

-Reality" is not inherent, but socially constructed. -A major focus is the study of how individuals and groups participate in the construction of a social reality. -Theorists believe there is no "natural" reason that constructs necessarily have to be the way they currently are in any specific culture. -Socialization is the dynamic, ongoing process by which an individual internalizes the values, beliefs, and norms of their society and learns to function as a member of that society (the passing along and picking these social constructs as we go along ... but this is a problem because then we view it as truth and internalize it). -Socialization helps to explain how social constructs are maintained, reaffirmed, and passed along to future generations. -Can be either a macro- or micro-level theory of society -Involves social constructs

Social condition

-Social determinants of health such as availability of food supplies, drug use, access to quality education, unemployment, crime rates, and access to healthcare. *Favorable conditional factors improve the overall quality of life. *Social problems are social conditions with a negative impact. *Social isolation is the complete or near-complete lack of contact with others in society.

Socialism vs. Communism

-Socialism is a social and political philosophy based on the belief that democratic means should be used to distributed wealth evenly throughout a society. Communism is characterized by a centrally based economy with all economic and political power resting in the hands of the central govt. -communism is the economic system where all businesses are owned by the people. Socialism is an economic system that also has all businesses owned by the people but it is not shared equally like communism.

Social institution

-Standardized sets of social norms organized to preserve a societal value -They systematically preserve the collective ethic of a society. Stability of these institutions help guard against anomie, which occurs when societal values do not adequately guide individual or group behavior. -They include: media, Education, Family, Religion, Government, economy, and politics, Health and medicine

Social epidemiology

-Study of the social determinants of health and the use of social concepts to explain patterns of health in the population. 1. What specific features of societal conditions affect health? 2. Through which pathways do societal conditions affect health?

Heritability

-The PROPORTION of variation among individuals that we can attribute to genes. The heritability of a trait may vary, depending on the range of populations and environments studied. -heritability is defined as the proportion of observable differences in phenotype attributable to genetic causes

absolute threshold

-The amount of stimulus energy needed for a person to perceive it at all -The weakest amount of a stimulus that a person can detect half the time -The minimum stimulation needed to detect a particular stimulus 50 percent of the time

construct validity

-The extent to which variables measure what they are supposed to measure -The extent to which a construct or test measures what it purports to measure. This could be potentially compromised by the use of self-report measures

Gestalt Psychology

-The law of similarity: the tendency to perceive objects of a similar size, shape, or color as a unit or figure. -The law of closure: the tendency to fill in the gaps in an incomplete image. -The law of good continuation: the tendency to group elements that appear to follow in the same direction as a single unit or figure. -The law of proximity: the tendency to perceive objects that are close to one another as a single unit. -Law of Prägnanz/law of simplicity: when several perceptual organizations of an assortment of visual elements are possible, the perceptual interpretation that occurs will be the one that produces the "best, simplest, and most stable shape" (three overlapping squares rather than as two six-sided objects and one four-sided object.)

difference threshold

-The minimum difference between two stimuli required for detection 50 percent of the time -The minimum amount of difference that can be detected between two stimuli -The smallest detectable difference between two stimuli (The just noticeable difference (JND) is the same value as the difference threshold. For example, a difference threshold of two pounds would equal one JND, four pounds would equal two JNDs, etc.)

Looking glass self

-The self is shaped by others and interaction with others and perception of others. -Individuals shape themselves on this perception, tending to confirm expectations. -Symbolic Interactionist Charles Cooley asserted that one's concept of oneself is developed in three stages: -We imagine how we must appear to others. -We imagine what others must think about us based on their observations of us. -We develop our feelings about ourselves based on our (truly or falsely) imagined judgments of others. This can cause us to behave in ways that confirm these imagined judgements.

just noticeable difference (JND)

-The smallest change in the intensity of a stimulus that we can detect -The minimal change in a stimulus that can just barely be detected (The just noticeable difference (JND) is the same value as the difference threshold. For example, a difference threshold of two pounds would equal one JND, four pounds would equal two JNDs, etc.)

Representativeness heuristic

-a mental shortcut whereby people classify something according to how similar it is to a typical case -judging the likelihood of things in terms of how well they seem to represent, or match, particular prototypes; may lead us to ignore other relevant information

histrionic personality disorder

-a personality disorder characterized by excessive emotionality and preoccupation with being the center of attention; emotional shallowness; overly dramatic behavior -characterized by excessive emotionality and attention-seeking, is more often explained in terms of children having learned to behave dramatically in order to gain attention from distant parents

primacy effect vs recency effect vs serial position effect

-primacy effect: (highest recall) refers to the better memory people have for items presented at the beginning of the list -recency effect: (2nd highest recall) refers to the better memory people have for most recent items, the ones at the end of the list -serial position effect: our tendency to recall best the last (a recency effect) and first items (a primacy effect) in a list

manifest functions vs latent functions

-the intended beneficial consequences of people's actions -Intended or obvious functions (Going to school to get an education and get a job) -the unrecognized and unintended consequences of any social pattern -Unitended or less recognizable functions' (Going to school and building relationships and meeting your future husband)

A not B error

-the tendency to reach for a hidden object where it was last found rather than in the new location where it was last hidden -In an A-not-B error, the individual perseveres in looking for an object in a location in which it was previously placed, even with the knowledge that it has been placed elsewhere. This is most common in infants under 12 months of age

iron law of oligarchy

-theory that power increasingly becomes concentrated in the hands of a few members of any organization -tendency of organizations to become increasingly dominated by small groups of people -A government ruled by a few powerful people -regardless of how democratic an organization is, all organizations will eventually and inevitably end up with a majority of the power in the hands of a few, thus leading to an oligarchy. The iron law of oligarchy suggests that democracy is practically and theoretically impossible, particularly in large and complex organizations

Tokenism

-when a company appoints a small group of women or minorities to high-profile positions, rather than more aggressively seeking full representation for that group -the practice of making only a perfunctory or symbolic effort to do a particular thing, especially by recruiting a small number of people from underrepresented groups in order to give the appearance of sexual or racial equality within a workforce. -When a single member of a minority group is present in an office, workplace, or classroom and is seen as a representative of that minority group rather than as an individual -refers to an organization's inadequate remediation of problems of inequality while exploiting minimal improvements. Such an approach is likened to giving a low-cost token to equality advocates in order to quell their demands for substantial reform.

Personality Disorder Clusters

A: Paranoid, schizoid, schizotypal B: Antisocial, borderline, histrionic, narcissistic C: Avoidant, dependent, obsessive compulsive

Absolute vs relative poverty

Absolute: socioeconomic condition in which people do not have enough money or resources to maintain a quality of living that includes basic necessities Relative: in which one is poor in comparison to the larger population (Absolute poverty is a measure of the bare minimum for subsistence, and the global level hovers between $1-2/day. However, one disadvantage of this is that poverty thresholds in developed nations tend to be higher than those in developing nations. For this reason, it can help to have nation-specific poverty thresholds, and to define absolute poverty as the minimum amount of money needed to secure basic life necessities. In 2010, this number was $22,113. Relative poverty, on the other hand, is a measure of poverty where family income is compared to the incomes of other families in the surrounding area.)

Locus of control

Beliefs about having or lacking control

self-efficacy

Beliefs in one's competency

Binocular depth cues

Binocular cues: Distance or depth cues that require the use of both eyes. -Disparity: the difference in image location of an object seen by the left and right eyes. When the two retinal images are very different, we interpret the object as being close by. When the two retinal images are more nearly identical, the object is perceived as being farther away -Convergence: the degree to which muscles rotate your eyes to focus on an object. The more the eyes converge, or rotate inward, to focus on an object, the greater the strength of the muscle signals and the closer the object is perceived to be.2. Binocular

Freud - Defense mechanisms

Clash of id and superego Denial - involves blocking external events from awareness. If some situation is just too much to handle the person just refuses to experience it Main defense mechanisms: 1. repression - ego's way of forcing undesired thoughts and urges to unconscious 2. suppression - deliberate, conscious form of forgetting 3. regression - faced with stress, older children return to earlier behaviors like thumb-sucking, throwing temper tantrums, clinging to their mothers (coping mechanisms that are characteristic of an earlier point in development and is an immature ego defense) 4. reaction formation - individuals suppress urges by unconsciously converting them into their exact opposites 5. projection - attribute their undesired feelings to others 6. rationalization - justification of behaviors in a manner that is acceptable to the self and society, logically explaining actions performed for different reasons in an effort to avoid blame and is an immature ego defense 7. displacement - describes transference of undesired urge from one person or object to another 8. sublimation - transformation of unacceptable urges into socially accepted behavior (Suppression is the act of VOLUNTARILY withholding an idea or feeling from conscious awareness. This is, perhaps, most easily confused with repression, which involves INVOLUNTARILY withholding an idea or feeling from conscious awareness, which is an immature ego defense)

What types of cultures exist in a society?

Generally, societies consist of a: -dominant culture: the group whose members are in the majority or who wield more power than other groups. -subculture: a group that lives differently from, but not opposed to, the dominant culture. A subculture is a culture within a culture. -counterculture is a subculture that opposes the dominant culture

Anxiety

Mental Disorder

Depressive disorders

Mental Disorder

Dissociative disorders

Mental Disorder

Neurocognitive disorders

Mental Disorder

Neurodevelopmental disorders

Mental Disorder

Obsessive-compulsive disorders

Mental Disorder

Schizophrenia spectrum and psychotic disorders

Mental Disorder

Sleep wake disorders (dyssomnias vs parasomnias)

Mental Disorder

Somatic symptom disorders

Mental Disorder

Specific Anxiety Disorders (panic disorder vs phobia vs generalized anxiety disorder vs social anxiety disorder)

Mental Disorder

Substance-related and addictive disorders

Mental Disorder

Trauma and stressor related disorders

Mental Disorder

Bipolar disorder

Mental Disorder (2nd most severe mental disorder)

Alzherimer's disease

Mental Disorder (Neurocognitive disorders)

Parkinson's disease

Mental Disorder (Neurocognitive disorders)

Autism Spectrum Disorder

Mental Disorder (Neurodevelopmental disorders)

Schizophrenia (positive, negative, and cognitive symptoms)

Mental Disorder (most severe mental disorder)

Monocular Cues

Monocular cues: the depth cues that depend on information that is available to either eye alone Relative Size: if objects are assumed to be the same size, the one that casts the smaller image on the retina appears more distant Interposition: if one object blocks the view of another, we perceive it as closer Relative Clarity: we perceive hazy objects as being more distant than sharp, clear objects Texture Gradient: change from a coarse, distinct texture to a fine, indistinct texture indicates increasing distance Relative Height: we perceive objects that are higher in the visual field as farther away Linear Perspective: parallel lines appear to converge as distance increases. The greater the convergence, the greater the perceived distance Light and Shadow: closer objects reflect more light than distant objects. The dimmer of two identical objects will seem farther away Relative Motion: as we move, stable objects appear to move as well. Objects that are near to us appear to move faster than objects that are farther away

Schatner-Singer Theory of Emotion

Must have both physical arousal and a cognitive label to experience emotion, can't have one w/o the other

Humanist Perspective

Theories of personality

Life course perspective

Theories of personality

Observational Learning

Theories of personality

Psychoanalytic perspective

Theories of personality

Social-cognitive perspective

Theories of personality

Trait Perspective

Theories of personality (OCEAN)

Erikson's Psychosocial Stages of Development

Theories of personality?

Freud's Psychosexual Stages

Theories of personality?

A person sees a small grassy area containing flowers outside of a residential building. This person's brain draws on various memories of gardens and the scents of flowers and grass in order to interpret that this grassy area is a garden. This is an example of: A. bottom-up processing. B. top-down processing C. superimposition. D. contextual reception.

Top-down processing is a Gestalt psychology term for interpreting sensory stimuli based on experience and larger contextual information in order to slowly work down and figure out the precise nature of each stimulus (choice B is correct). Bottom-up processing is the Gestalt term for the reverse process, by which one starts with a small sensory stimulus and integrates progressively more contextual information in order to determine its nature (choice A is wrong). Superimposition is a process whereby one stimulus is layered on top of another (choice C is wrong). Contextual reception is simply another way of saying that one is receiving contextual stimuli, not necessarily even processing it (choice D is wrong).

Macro sociology

Top-down view Begins with society as a whole Focuses on large-scale social structures and drills down to how these structures affect groups/individuals Premise: individual positions within social structures determine behavior. (E.g. education, healthcare, government)

Many forces influence how people relate to one another, become socialized beings, and learn to develop relationships. Which of the following is NOT one of the agents of socialization? A. Family B. School C. Travel D. Television

While travel can be highly educational in terms of introducing people to other cultures and ways of life, it is not specifically an agent of socialization; it does not necessarily help people learn how to relate, as one can travel alone and avoid contact with others (choice C is not an agent of socialization and is therefore correct). Family is one of the first and primary sources of socialization from which a person learns how to relate to others --- one's parents, siblings, and extended family (depending on the family structure). Lessons learned from the family often form the basis for many future relationship choices (choice A is an agent of socialization and is therefore wrong). School is another primary source of socialization, as teachers try to educate children about the values of the specific culture and the important skills needed to survive in that culture (choice B is an agent of socialization and is therefore wrong). Television, along with other forms of mass media, provide people with a glimpse of how to relate to the larger world, introducing people to other lifestyles as well as popular trends and ideas (choice D is also an agent of socialization and is therefore wrong).

Mental set

a tendency to approach a problem in one particular way, often a way that has been successful in the past

regression coefficient

a way to characterize the relationship between the independent and dependent variables, and graphically is represented by the slope of the line that represents the rate of change of one variable as a function of changes in the other

iconic vs echoic memory

iconic (visual) less than 1 second echoic (auditory) around 2 - 4 seconds

compliance (low ball technique vs ingratiation technique vs norm of reciprocity)

low ball = The low-ball technique involves making an attractive offer and then changing the terms of the deal (e.g., quoting Jack a $100 travel package for his special fishing trip, then later informing him that substantial additional fees will be required)

Sensory information first arrives at the

thalamus!

Globalization

the processes of international integration It has been proposed that the internet, by allowing the spread of information, has increased the ability of individuals to empathize with people in other countries, which has had a profound impact on international donations. A contemporary economic sociologist might use which one of the following concepts to discuss this relationship between social change and charitable acts? Globalization

Functional fixedness

the tendency to think of things only in terms of their usual functions; an impediment to problem solving

Linguistic relativity hypothesis

the theory that thought processes and concepts are controlled by language


Related study sets

Med Law and Ethics - Mod 11 - Compliance in Healthcare

View Set

INFO 323 - Comprehensive Final Exam

View Set

AP Gov, 4th Quarter, Unit 1, Chapter 15 The Federal Courts, Woll: the Federal Judiciary

View Set

Modules 11 - 13: IP Addressing Exam

View Set

17. English Phrasal Verbs in Use Advanced, Unit 17, Time

View Set

Sample Behavioral Interview Questions

View Set

Fundamentals: Infection Prevention - Clinical Questions

View Set